Vilenkin in relation to Kalam Cosmological Argument

frankie609
Posts: 6
Joined: 2011-02-08
User is offlineOffline
Vilenkin in relation to Kalam Cosmological Argument

Hi everyone!

 

Ignoramus here. I hope you have the time to answer my questions.

 

Part I

Vilenkin said in his book:

"It is said that an argument is what convinces reasonable men and a proof is what it takes to convince even an unreasonable man. With the proof now in place, cosmologists can no longer hide behind the possibility of a past-eternal universe. There is no escape, they have to face the problem of a cosmic beginning."

(Vilenkin, Many Worlds in One, 176)

1) What does it exactly mean when he says "cosmic beginning"?

2) What are his arguments the led him to this conclusion?

3) Does this imply that the universe really doesn't exist before the big bang? That the universe really had a beginning?

 

Part II

I've been hearing different models of cosmology and such include Vilenkin's model.

1) Are they just simply hypotheses? Or could they be categorized as a valid scientific theory?

 

Part III

Reading upon his paper "Inflationary spacetimes are not past-complete"

http://arxiv.org/PS_cache/gr-qc/pdf/0110/0110012v2.pdf

1) What exactly is the message of this paper? Does it tell us that the universe need to have a beginning?

2) How credible is this paper? Is it valid hypothesis? Is it a valid scientific theory?

 

Can somebody please enlighten me on this matter? I am lost.

 

Thanks and much appreciated.


BobSpence
High Level DonorRational VIP!ScientistWebsite Admin
BobSpence's picture
Posts: 5939
Joined: 2006-02-14
User is offlineOffline
Inflationary/expansionary/"Bi

Inflationary/expansionary/"Big Bang" models do not rule out an indefinite past for the Universe in the broader sense, at all, just for our subset of ultimate reality, of the "Metaverse".

There are models which have new "Big Bangs" seeded out of the fragments of the "Big Rip", the ultimate ever-accelerating rate of expansion.

'Reality" could quite easily consist of an endless sequence or series of Big Bangs.

Or space-time could be finite and boundless, curving back on itself in a higher dimensionality.

I had a look at that last paper, and it would take a bit more time than I currently have to come to grips with what they are driving at.

The Kalam argument proves nothing at all about 'God' it just makes an enormous and un-justified leap from a "first cause" to that "first cause" having all the attributes of whatever 'God' the arguer has in mind.

From the study of particle Physics and Quantum Theory we know that Planck-scale events do NOT necessarily have a 'cause', at least in any simplistic sense, but even if they do, we also know that 'causes' do not necessarily have to be 'greater' in whatever sense than what they 'cause'.

Any even happens because of two things:

1. The 'state' of all the relevant parts of existence close enough to be involved,  which means that the event is physically and energetically possible, and

2. The occurrence of some 'trigger' event that finally pushes the parts of reality involved over some threshold.

Example, an avalanche requires a large accumulation of snow on a slope to the point that the 'stickiness' of the snow is only just sufficient to hold it together (1). Then, that last falling snowflake, or an external vibration, takes it over that threshold, and all the accumulated gravitational potential energy is turned into kinetic energy of motion (2).

With unstable ( 'radio-active' ) atoms, the energy of all the particles in the post-decay nucleus plus that of the emitted particle needs to be just slightly less than that of the original nucleus, otherwise it would violate the principle of Conservation of Energy and the Second Law: something cannot spontaneously go to a higher energy state without an external input. There has to be some intermediate state that has slightly higher energy than that of the bound nucleus, which prevents it from decaying immediately, that holds it together, like a slight ridge at the top of a slope which is just enough to stop a stationary ball from rolling down.

Thanks to the Uncertainty principle, the exact energy of the system cannot be determined or defined, only a probability distribution, a 'wave-function'. This allows us to compute the probability that the system will temporarily have sufficient energy to exceed the threshold within any specified time period, which can be used to calculate how long we would have to wait until that probability was 50%, giving us the 'half-life' of that particular combination of particles, ie, that isotope of that element.

The question is, what 'causes' a particular atom to decay at a particular instant?

Is it a pure, elemental 'randomness' at the heart of physical reality, or is that apparent randomness the outcome of some underlying 'deterministic' process, such as an underlying 'sea' of interacting particles or energy packets? Such as the way the timing and velocity of the next oxygen molecule from the atmosphere of our planet to hit a specified area of a surface is as close to effectively random, as unpredictable, as anything could be, yet that does not imply or require that the interactions of the molecules has any non-deterministic component.

 

Just trying to point out that the extremely simplistic medieval notion of 'causality' embodied in the KCA totally fails to reflect our current understanding.

And then, of course, there is zero implication or 'need' for a sentient decision-maker to be that 'cause' - just some random or quasi-random quality of reality.

Is 'God' personally telling each radioactive atom when to decay? Busy guy, no wonder he appears to have no time to fix all the problems and suffering he caused by his ham-fisted design of us and our evironment...

Favorite oxymorons: Gospel Truth, Rational Supernaturalist, Business Ethics, Christian Morality

"Theology is now little more than a branch of human ignorance. Indeed, it is ignorance with wings." - Sam Harris

The path to Truth lies via careful study of reality, not the dreams of our fallible minds - me

From the sublime to the ridiculous: Science -> Philosophy -> Theology


harleysportster
atheist
harleysportster's picture
Posts: 3359
Joined: 2010-10-17
User is offlineOffline
Why the Kalam Argument Fails

“It is proof of a base and low mind for one to wish to think with the masses or majority, merely because the majority is the majority. Truth does not change because it is, or is not, believed by a majority of the people.”
― Giordano Bruno


frankie609
Posts: 6
Joined: 2011-02-08
User is offlineOffline
Thanks Bob, I'm willing to

Thanks Bob, I'm willing to wait till you look at that paper. Much appreciated.

I was arguing with someone and it seems to me he's giving me the impression that, hey! they're scientists too! they can't be wrong can they? I'm not even sure if that what exactly the author means when they say "the universe had a beginning".


frankie609
Posts: 6
Joined: 2011-02-08
User is offlineOffline
 Hi harleysportster, I

 Hi harleysportster, I appreciate your links. Thanks.


harleysportster
atheist
harleysportster's picture
Posts: 3359
Joined: 2010-10-17
User is offlineOffline
frankie609 wrote: I was

frankie609 wrote:

 

I was arguing with someone and it seems to me he's giving me the impression that, hey! they're scientists too! they can't be wrong can they? I'm not even sure if that what exactly the author means when they say "the universe had a beginning".

The notion that the universe had to have a first cause is where the argument falls flat. If they believe that god or a creator is behind the universe, then logically, they would have to explain a first cause or a beginning for god. Yet, most theists argue that god was always there. It's an inherent contradiction.

“It is proof of a base and low mind for one to wish to think with the masses or majority, merely because the majority is the majority. Truth does not change because it is, or is not, believed by a majority of the people.”
― Giordano Bruno


BobSpence
High Level DonorRational VIP!ScientistWebsite Admin
BobSpence's picture
Posts: 5939
Joined: 2006-02-14
User is offlineOffline
Yeah, the KCA has no force

Yeah, the KCA has no force whatever as argument for God.

It states what may be a simple 'truism' - the need for a 'cause', then makes a naked assertion that only 'God' could be that 'cause'.

The other fail in causality arguments is that even if everything needs a cause, then as long as the 'magnitude' and duration of each cause gets smaller than its 'effect', then even a notionally infinite sequence of cause-effect steps may involve only a finite energy and duration. This follows from the maths of summation of infinite series, which was not worked out at the time these arguments were made. The classic example is that

Sn = 1 + r1 + r2 + r3 + r4 + ... + rrn-1 + rn = ( 1 - rn )/( 1 - r)

If r is less than 1, rn => zero as n => infinity, so Sinf = 1/(1-r);

if r = 1/2, Sinf = 2:

1 + 1/2 + 1/4 + 1/8 + 1/16 +.... = 2

 

Favorite oxymorons: Gospel Truth, Rational Supernaturalist, Business Ethics, Christian Morality

"Theology is now little more than a branch of human ignorance. Indeed, it is ignorance with wings." - Sam Harris

The path to Truth lies via careful study of reality, not the dreams of our fallible minds - me

From the sublime to the ridiculous: Science -> Philosophy -> Theology


BobSpence
High Level DonorRational VIP!ScientistWebsite Admin
BobSpence's picture
Posts: 5939
Joined: 2006-02-14
User is offlineOffline
Ok, if I have time.Just note

Quote:

Thanks Bob, I'm willing to wait till you look at that paper. Much appreciated.

I was arguing with someone and it seems to me he's giving me the impression that, hey! they're scientists too! they can't be wrong can they? I'm not even sure if that what exactly the author means when they say "the universe had a beginning".

Ok, if I have time.

Just note that none of this stuff makes the KCA in anyway a valid argument, because it totally fails to provide any more than naked assertion that any 'first cause' can only be 'God'.

So forget the KCA nonsense.

But if you are really interested in the origin and nature of the Universe, that's fine.

Favorite oxymorons: Gospel Truth, Rational Supernaturalist, Business Ethics, Christian Morality

"Theology is now little more than a branch of human ignorance. Indeed, it is ignorance with wings." - Sam Harris

The path to Truth lies via careful study of reality, not the dreams of our fallible minds - me

From the sublime to the ridiculous: Science -> Philosophy -> Theology


frankie609
Posts: 6
Joined: 2011-02-08
User is offlineOffline
RE: KCA

"And then, of course, there is zero implication or 'need' for a sentient decision-maker to be that 'cause' - just some random or quasi-random quality of reality."

@BobSpence1

I totally agree with that argument. However I don't even want him to get past the first two premises which state that the universe must have a cause and that the universe had a beginning. He just cited Vilenkin's paper which of course, somebody like me would take a hard time whether to agree with that paper to its claim (if it really claims that the universe must have a finite past) if I don't know the credibility of that paper.

Is it true that it is a common consensus in the scientific community that the universe had a beginning? If it is, what do they mean by beginning? A temporal beginning? Is it some sort of an absolute beginning of everything? Or does it mean something else the way nothing might not literally mean an absolute nothingness or perhaps an empty space?

Crap, I should have majored in Physics LOL

 


Answers in Gene...
High Level Donor
Answers in Gene Simmons's picture
Posts: 4214
Joined: 2008-11-11
User is offlineOffline
 OK, I read the paper last

 

OK, I read the paper last night.

For the record, it provides no support whatsoever to the universe having been created in the manner of the Kalam argument. The person you are arguing with either:

 

A: did not bother to read the paper before using it as a source or

 

B: did struggle through the paper but only with such a limited grasp of the concepts that he was unable to see the basic ideas which were presented.

 

Past that, I am not really clear on what you mean by the idea that the argument and the concept presented in the paper could be “not valid” on a scientific basis.

 

When I hear of something as being invalid in that way, the first thing which comes to mind is that it might be bad speculation dressed up as science. Pretty much stuff like UFO conspiracy crap and the like. This paper is definitely not that.

 

What the paper actually is is worth noting. Science has various ways of sniffing out bad work. This paper passes sufficient muster to be worth considering as having scientific value.

 

A: It was co-authored by three respectable scientists. Granted, you would have to be quite into science in order to know their names but they are the real deal.

 

B: It was submitted for peer review and passed muster on that account.

 

C: It was published in a recognized major source for such work.

 

Now that having been said, the confusion may lay in the idea of whether or not the paper is “right”. Honestly, science does not work on the basis of proving something to be right in the mathematical sense. Rather, what would matter would be if it seemed so right that it was very unlikely to be wrong.

 

Could this paper be at least that much right? Sure it could. However, it is also out on the bleeding edge of science where there are as many ideas of what may be right as there are people working in thie field.

 

They cannot all be right and there is even no good reason on that level why any of them have to be right. Perhaps the right idea is one that has not been discovered just yet. However, what that idea may well be will only be known if today's physicists keep throwing out best guesses and submitting for peer review. In layman's terms, peer review is quite like scientists comparing stuff with each other and asking the basic question “Does this look right to you?”

NoMoreCrazyPeople wrote:
Never ever did I say enything about free, I said "free."

=


frankie609
Posts: 6
Joined: 2011-02-08
User is offlineOffline
 @Answers in Gene

 @Answers in Gene Simmons

 

Thanks, that is very informative. Smiling

One series of questions though. Does the paper really tell us that the universe had a beginning? And what about the multiverse that contains the universes? Does the paper clearly tell us that Vilenkin was referring to a universe and not to some sort of a multiverse in which the word universe doesn't necessarily mean "everything in reality that exists"?

Someone helped me out and referred this link to clarify what the word universe might mean depending on the context.

 

"There was a time when "universe" meant "all there is." Everything. The whole shebang. The notion of more than one universe, more than one everything, would seemingly be a contradiction in terms. Yet a range of theoretical developments has gradually qualified the interpretation of "universe." To a physicist, the word's meaning now largely depends on context. Sometimes "universe" still connotes absolutely everything. Sometimes it refers only to those parts of everything that someone such as you or I could, in principle, have access to. Sometimes it's applied to separate realms, ones that are partly or fully, temporarily or permanently, inaccessible to us; in this sense, the word relegates ours to membership in a large, perhaps infinitely large, collection."

- http://www.squidoo.com/brian-greene-the-hidden-reality

 

 


Answers in Gene...
High Level Donor
Answers in Gene Simmons's picture
Posts: 4214
Joined: 2008-11-11
User is offlineOffline
 frankie609 wrote:One

 

frankie609 wrote:
One series of questions though. Does the paper really tell us that the universe had a beginning?

 

Actually no it does not. What it is saying is pretty much that there is a way of thinking about something that might be called a universe and it might be a decent model for our own, or not. Now if it is close enough to our universe, then our universe would not extend infinitely back in time.

 

However, just because it may not do that is not the same as the universe having had a beginning. What it is doing is creating a mathematical model of what would be known as finite in extent but infinite in boundary. OK, that is going to be kind of hard to grasp without dumping the math on you but there are lots of mathematical models that are of that nature.

 

Before I go on to your next question though, let me lay some groundwork. As the blog that you linked to properly says, the definition of the word “universe” has changed over time. However, the blog does not really cover the extent of the change as well as I would like for you to consider.

 

To Plato, the universe was the Earth and the Planets. Then too, he thought that the sun and moon were planets, mainly because he did not have that kind of background.

 

Later on, in the last half of the 1700's, astronomers finally learned how to measure the distance to the stars. Then the idea of the universe was extended to cover those objects as well.

 

A century ago, when Einstein was doing his most notable work, the universe was considered to be the whole galaxy. Of course we did not, at that time know about other galaxies just yet. But we had a concept down that the assemblage of stars was finite and there were no stars more than a hundred thousand light years away.

 

In the late 1920's, Hubble (yes the guy the telescope was named for) was able to show that certain “clouds” of gas were actually other galaxies. Still, we had no clue how many there might be or if there could be an infinte number that just went on forever.

 

Then in the 1930's Hubble went on to show that the universe was expanding. Now we are really running into a possibly huge problem here. There must be some distance at which galaxies can no longer be visible. They would still have originated in the big bang but as they are moving away from us faster than the speed of light, they are forever isolated from us. We will never seem them or interact with them in any way. So are they part of our universe and in what sense can that even be a meaningful question.

 

frankie609 wrote:
Does the paper clearly tell us that Vilenkin was referring to a universe and not to some sort of a multiverse in which the word universe doesn't necessarily mean "everything in reality that exists"?

 

OK, again, that paper is only in reference to a model that might be a way to think about the stuff that came from the big bang. It still would not reference the Kalam Argument.

 

 

I happen to be about three quarters of the way through that book. I will have to get back to you on the details but he is discussing several ideas for a “multiverse” and asking what they might mean to our understanding of matters. Basically, as far into the book as I am, all concepts of a multiverse would be permanently inaccessible to us. However, if any of them are correct, then there would be implications as far as how the stuff that we can know about must exist and we may, one day, find evidence that we can measure and understand that would point to meaningful answers about a possible “multiverse”.

NoMoreCrazyPeople wrote:
Never ever did I say enything about free, I said "free."

=


frankie609
Posts: 6
Joined: 2011-02-08
User is offlineOffline
@Answers in Gene

@Answers in Gene Simmons

Really sorry for this, I'm complete ignorant on this and need an answer that I could use in a layman's term.

Answers in Gene Simmons wrote:
 

Actually no it does not. What it is saying is pretty much that there is a way of thinking about something that might be called a universe and it might be a decent model for our own, or not. Now if it is close enough to our universe, then our universe would not extend infinitely back in time.

 

What do you mean it doesn't? What does Vilenkin mean by cosmic beginning in "There is no escape, they have to face the problem of a cosmic beginning"? And what does he mean by past-eternal universe is impossible? Was he referring only to our universe or are all universes like that?

 

I don't have a copy of his book so it would be really hard for me to make any conclusion about his work but get some inputs from those who have read it.

 

Answers in Gene Simmons wrote:
 

However, just because it may not do that is not the same as the universe having had a beginning.

 

You mean, if our universe couldn't have an infinite past it still doesn't mean it had a beginning? Just what does the word beginning mean in this context? How could it be possible for a universe not to have a beginning while having a non infinite past? 

 

 

Answers in Gene Simmons wrote:
 

Then in the 1930's Hubble went on to show that the universe was expanding. Now we are really running into a possibly huge problem here. There must be some distance at which galaxies can no longer be visible. They would still have originated in the big bang but as they are moving away from us faster than the speed of light, they are forever isolated from us. We will never seem them or interact with them in any way. So are they part of our universe and in what sense can that even be a meaningful question.

 

Very interesting. It didn't occurred to me that there's something that could travel faster than speed of light. I thought such things would never exist.

 


Ktulu
atheist
Posts: 1831
Joined: 2010-12-21
User is offlineOffline
frankie609 wrote:Very

frankie609 wrote:

Very interesting. It didn't occurred to me that there's something that could travel faster than speed of light. I thought such things would never exist.

 

Gene would probably explain this better then I would.  Nothing moves faster then the speed of light, however, since space-time is expanding that makes it appear to us as if it is moving faster then the speed of light.  So if galaxy X is moving relative to us at 80% the speed of light, as space expands between us, it will compensate for the remaining 21% to make it faster then the speed of light.  Once that critical 101% speed has been reached the light from that galaxy will no longer be able to reach us.  Since the universe expansion is accelerating due to dark energy, eventually the night sky will be completely dark.  Of course there won't be a night sky as we understand it since Earth won't be around then.

I was reading an interesting article on the physics.org (or maybe sciam.com) website a few years back.  It was regarding a hypothetical future civilization that would no longer be able to observe any other galaxy.  It was question if said civilization would be able to determine if the universe was expanding. 

"Don't seek these laws to understand. Only the mad can comprehend..." -- George Cosbuc


Answers in Gene...
High Level Donor
Answers in Gene Simmons's picture
Posts: 4214
Joined: 2008-11-11
User is offlineOffline
 OK, I wanted to take a

 

OK, I wanted to take a couple of days to think about the most recent parts of this thread, so if you were waiting on tenterhooks, well, sorry for that.

 

Anyway, the first thing that I would ask you to think about is that the paper on arXiv does not say anything like “this is the way that the universe must work and no other explanation can possibly make sense”.

 

It only advances a model for some thoughts about how the universe might work. Even then, there is nothing in the paper that pins down any new insight beyond some stuff that we are already working on from other lines of thought. It could well turn out years from now to be really wrong.

 

>>>>>>>>>>>>>>>>>>>>>>>>>>>>>>>>>>>>>>>>>>>>>>>>>>>>>>>>>>>>>>>>>>>

 

That being said, I too do not have a copy of Vilenkin's book. So I lack the context to know what the one quote is even about. Heck but I can't even verify that the quote is even in the book. That and if it is, it may have been pulled irresponsibly in an attempt to obscure the matter at hand.

 

Theists do that much all the time. It is called 'quote mining”.

 

>>>>>>>>>>>>>>>>>>>>>>>>>>>>>>>>>>>>>>>>>>>>>>>>>>>>>>>>>>>>>>>>>>>>

 

OK, now for the science, I am going to take things in the order that suits me in order to try to tell the story. Really though, it is really hard to tell a story that can only be fully understood by doing the math without actually doing the math. Even so, I will give it a shot.

 

>>>>>>>>>>>>>>>>>>>>>>>>>>>>>>>>>>>>>>>>>>>>>>>>>>>>>>>>>>>>>>>>>>>>

 

Let me start with the expansion of the universe. That much has been known since the 1930's. The whole deal with dark energy has only been known for about 12~15 years.

 

I am going to ask you to go back to before dark energy was known about. I am also going to ask you to think about a few distant galaxies.

 

  1. Galaxy A starts at one billion light years distant.

  2. Galaxy B starts at two billion light years distant.

  3. Galaxy C starts at three billion light years distant.

  4. Galaxy D starts at four billion light years distant.

  5. Galaxy E starts at five billion light years distant.

 

Now, in some amount of time (it doesn't matter how long), the extent of the universe doubles. Now how far away are the set of galaxies?

 

  1. Galaxy A is now at two billion light years distant.

  2. Galaxy B is now at four billion light years distant.

  3. Galaxy C is now at six billion light years distant.

  4. Galaxy D is now at eight billion light years distant.

  5. Galaxy E is now at ten billion light years distant.

 

So do you see what is going on here? The farther away a galaxy is when you start looking at it, the faster it must be moving away. This has nothing at all to do with the acceleration that might be provided by dark energy. It only has to do with the distance at some fixed moment in time.

 

For a galaxy which is far enough away, the distance that it would cover in that amount of time would be greater that the distance that a beam of light could have covered in the same amount of time. Thus, the light from that distant galaxy could never possibly reach us. The same could be said for the gravity or any other measurement that must be contingent on the speed of light.

 

For the purpose of understanding the nature of the universe, such distant galaxies might as well be inside of black holes. There is simply no thing at all that can ever be said about them because we can never know them.

 

>>>>>>>>>>>>>>>>>>>>>>>>>>>>>>>>>>>>>>>>>>>>>>>>>>>>>>>>>>>>>>>>>>>>>

 

Adding the acceleration due to dark energy does not modify the fact that there are galaxies that we can't know about or interact with.

 

>>>>>>>>>>>>>>>>>>>>>>>>>>>>>>>>>>>>>>>>>>>>>>>>>>>>>>>>>>>>>>>>>>>>>>>>

 

I think that that is enough for now. That is complicated enough and I propose that we get that much hammered out before we move on to the next concept.

NoMoreCrazyPeople wrote:
Never ever did I say enything about free, I said "free."

=


Ktulu
atheist
Posts: 1831
Joined: 2010-12-21
User is offlineOffline
 Thank you for clearing

 Thank you for clearing that concept out, it does make more sense now from the simplistic understanding I had prior to reading it.

"Don't seek these laws to understand. Only the mad can comprehend..." -- George Cosbuc


Answers in Gene...
High Level Donor
Answers in Gene Simmons's picture
Posts: 4214
Joined: 2008-11-11
User is offlineOffline
 Ah yes, happy to help

 

Ah yes, happy to help where I may.

 

One problem that keeps coming up in such threads is that people seem to want to jump in the deep end of the pool without dealing with the basic stuff that really should be dealt with first. That being said, a great deal of such material has much math behind it and getting that down in prose is not so easy.

 

Allow me to suggest that you read a book called “The Big Bang” by Simon Singh.

 

It starts with the question “how big is the Earth?” and moves on from there.

 

Do you know how easy it is to measure the size of the Earth? I would guess not.

 

The most important part is to have a big flat place with something tall in the general area. A public park with a flag pole would be ideal. Simple geometry is enough to establish how tall the flag pole is. Get that much down and we can move on from there.

NoMoreCrazyPeople wrote:
Never ever did I say enything about free, I said "free."

=


Ktulu
atheist
Posts: 1831
Joined: 2010-12-21
User is offlineOffline
I have a pretty good grasp

I have a pretty good grasp of physics for a layman, but my understanding is still ridden with holes and of course I lack knowledge of the math behind it all.  I'm fascinated by the concepts but the details sometimes escape me.  I've read a lot of layman oriented material and even attempt some expert oriented papers though most are over my head for the above mentioned reasons.  If I ever have time(read never will happen) I would like to take some advanced courses to clear my understanding.  

"Don't seek these laws to understand. Only the mad can comprehend..." -- George Cosbuc


Mr_Metaphysics (not verified)
Posts: 4294964976
Joined: 1969-12-31
User is offlineOffline
BobSpence1

(Deleted post.)


Mr_Metaphysics (not verified)
Posts: 4294964976
Joined: 1969-12-31
User is offlineOffline
Hi, I'm new to the forum. 

Hi, I'm new to the forum.  I am a theist, and I hope we can be civil in our discourse. 

BobSpence1 wrote:

Inflationary/expansionary/"Big Bang" models do not rule out an indefinite past for the Universe in the broader sense, at all, just for our subset of ultimate reality, of the "Metaverse".

There are models which have new "Big Bangs" seeded out of the fragments of the "Big Rip", the ultimate ever-accelerating rate of expansion.

'Reality" could quite easily consist of an endless sequence or series of Big Bangs.

You need to demonstrate how it is possible for there to be an endless sequence of anything, let alone Big Bangs.  I do not see how the idea of an actual infinite quantity is even logically coherent.

Quote:
The Kalam argument proves nothing at all about 'God' it just makes an enormous and un-justified leap from a "first cause" to that "first cause" having all the attributes of whatever 'God' the arguer has in mind.

'First cause' is not a benign notion; many things necessarily follow from the idea, such as eternity, timelessness, self-existence, and immateriality.

Quote:
From the study of particle Physics and Quantum Theory we know that Planck-scale events do NOT necessarily have a 'cause', at least in any simplistic sense, but even if they do, we also know that 'causes' do not necessarily have to be 'greater' in whatever sense than what they 'cause'.

My understanding is that there are about 10 different interpretations of QM, and not a single one claims that things pop into existence from nothing... not even virtual particles. 

Quote:
With unstable ( 'radio-active' ) atoms, the energy of all the particles in the post-decay nucleus plus that of the emitted particle needs to be just slightly less than that of the original nucleus, otherwise it would violate the principle of Conservation of Energy and the Second Law: something cannot spontaneously go to a higher energy state without an external input. There has to be some intermediate state that has slightly higher energy than that of the bound nucleus, which prevents it from decaying immediately, that holds it together, like a slight ridge at the top of a slope which is just enough to stop a stationary ball from rolling down.[

Thanks to the Uncertainty principle, the exact energy of the system cannot be determined or defined, only a probability distribution, a 'wave-function'. This allows us to compute the probability that the system will temporarily have sufficient energy to exceed the threshold within any specified time period, which can be used to calculate how long we would have to wait until that probability was 50%, giving us the 'half-life' of that particular combination of particles, ie, that isotope of that element.

Sorry, I do not understand any of this.  I'm not a physicist.  If you are looking for a response, then please rephrase in terms that I can understand.

Quote:
Just trying to point out that the extremely simplistic medieval notion of 'causality' embodied in the KCA totally fails to reflect our current understanding.

'Causality' simply means that one thing necessitates another thing-- nothing more, nothing less.  It's not some primitive medieval notion, and even in the Middle Ages I doubt there was consensus on this issue.  The law of causality simply trades on the idea that nothing cannot produce something.  From the limited research I've done on quantum mechanics, I really do not see how that is contradicted in any way.  (I think it's ridiculous to assume that natural science somehow managed to scientifically verify things about causality, as if causality itself is something to be tested scientifically; 'causality' is an issue of logic, not of science.


BobSpence
High Level DonorRational VIP!ScientistWebsite Admin
BobSpence's picture
Posts: 5939
Joined: 2006-02-14
User is offlineOffline
Vilenkin's paper seems to be

Vilenkin's paper seems to be written mainly in the context of the Big Bang, suggesting problems and possible solutions to ideas about the early stages of the expansion, in particular the idea of 'cosmic inflation',so I don't quite see it being that relevant to ideas of origin of the BB itself.

In any case, the problem with the KCA is not so much whether there was, or had to be, a 'cause', of everything, which even cosmologists argue over, and gets involved with definitions of Time itself.

It is the totally unjustified leap from a 'first cause' to an Abrahamic God. Or perhaps I should say, a 'justification' from some obsolete metaphysical assumptions about cause-and-effect, infinite regress, and the nature of 'mind'. Even with that, I still see no justification for why this assumed sentient being had to be infinite, or the standard of morality, or even necessarily care about us in any positive way.

 

 

Favorite oxymorons: Gospel Truth, Rational Supernaturalist, Business Ethics, Christian Morality

"Theology is now little more than a branch of human ignorance. Indeed, it is ignorance with wings." - Sam Harris

The path to Truth lies via careful study of reality, not the dreams of our fallible minds - me

From the sublime to the ridiculous: Science -> Philosophy -> Theology


Mr_Metaphysics (not verified)
Posts: 4294964976
Joined: 1969-12-31
User is offlineOffline
BobSpence1 wrote:It is the

BobSpence1 wrote:

It is the totally unjustified leap from a 'first cause' to an Abrahamic God. Or perhaps I should say, a 'justification' from some obsolete metaphysical assumptions about cause-and-effect, infinite regress, and the nature of 'mind'. Even with that, I still see no justification for why this assumed sentient being had to be infinite, or the standard of morality, or even necessarily care about us in any positive way.

Actually, the argument is rarely used to prove that any specific religion is right.  It merely establishes the existence of an infinite being--whether he actually cares about us is a matter of historical record.

I'm not quite getting where you derive this idea about 'obsolete metaphysical assumptions'.  What do you understand to be the primitive notion of cause and effect, and how is that any different than what we believe today?  I'll pose the same question with regard to an infinite regress and the mind.


BobSpence
High Level DonorRational VIP!ScientistWebsite Admin
BobSpence's picture
Posts: 5939
Joined: 2006-02-14
User is offlineOffline
Mr_Metaphysics

Mr_Metaphysics wrote:

BobSpence1 wrote:

It is the totally unjustified leap from a 'first cause' to an Abrahamic God. Or perhaps I should say, a 'justification' from some obsolete metaphysical assumptions about cause-and-effect, infinite regress, and the nature of 'mind'. Even with that, I still see no justification for why this assumed sentient being had to be infinite, or the standard of morality, or even necessarily care about us in any positive way.

Actually, the argument is rarely used to prove that any specific religion is right.  It merely establishes the existence of an infinite being--whether he actually cares about us is a matter of historical record.

I'm not quite getting where you derive this idea about 'obsolete metaphysical assumptions'.  What do you understand to be the primitive notion of cause and effect, and how is that any different than what we believe today?  I'll pose the same question with regard to an infinite regress and the mind.

The 'cause' of some event can often be an indefinitely large number of contributing factors, with some 'proximate' relatively trivial event triggering a far larger event, like the final snowflake landing on an unstable pile-up of snow on a mountain slope. The 'thinking' behind ideas like the 'principle of sufficient reason' is just so obsolete and out of touch with the modern understanding. Modern Complexity Theory has gone utterly beyond any of those ideas, the implications what can emerge from sufficiently complex deterministic systems.

Then there is the simplistic understanding of 'determinism' and what they thought it implied. The idea of chaos due to feedback in non-linear systems, leading to situations popularly characterised as the 'Butterfly Effect' are related, but add a dimension where a difference in magnitude or direction of some event that can literally approach zero may determine which of two or more trajectories some fully deterministic process may take that can lead to dramatically different large-scale outcomes. This means that fully deterministic systems can behave in an absolutely unpredictable way. 

Of course they didn't 'get' feed-back, as it is understood today, at all.

Then there was the old idea that all 'motion' required a continuing 'mover', whereas only change of motion requires a cause.

And motion is in fact 'energy', which is a fundamental of existence, only changing its form, so 'motion' is always with us, and absolutely does not need a 'mover'.

Then you throw in Quantum Mechanics, which makes randomness and uncertainty fundamentals of reality, and then there is Einstein's Relativity, of course.

And the failure to grasp that matter is in no way 'mere matter', it is a higher form of 'condensed' energy, absolutely required for any persistent structure needed in turn to support complex processes, such as living organisms and cognition.

They had naive ideas on 'infinity' and infinite summation ( Zeno's 'paradoxes' ).

Infinite regress only leads to any real infinities if it is assumed the 'causes' are consistently greater in some sense that their effect. 

The sum of an infinite number of finite quantities, such as 1 + 1/2 + 1/4 + 1/8 +... = 2.0, ie, quite finite.

The picture we have now of the nature of mind and its dependence on the brain, emerging from many, many studies presents a far different picture from what was assumed then.

The very idea that intuition and introspection and 'pure' reason were all that were needed to grasp 'truth' is a basic fallacy underlying much of this obsolete thinking, the often expressed contempt for 'empiricism', which persists even today in many sub-cultures.

The lack of understanding of all these ideas means their speculations are worthless, their arguments void.

IOW, they were wrong on just about everything that mattered.

Favorite oxymorons: Gospel Truth, Rational Supernaturalist, Business Ethics, Christian Morality

"Theology is now little more than a branch of human ignorance. Indeed, it is ignorance with wings." - Sam Harris

The path to Truth lies via careful study of reality, not the dreams of our fallible minds - me

From the sublime to the ridiculous: Science -> Philosophy -> Theology


Mr_Metaphysics (not verified)
Posts: 4294964976
Joined: 1969-12-31
User is offlineOffline
BobSpence1

BobSpence1 wrote:

Mr_Metaphysics wrote:

BobSpence1 wrote:

It is the totally unjustified leap from a 'first cause' to an Abrahamic God. Or perhaps I should say, a 'justification' from some obsolete metaphysical assumptions about cause-and-effect, infinite regress, and the nature of 'mind'. Even with that, I still see no justification for why this assumed sentient being had to be infinite, or the standard of morality, or even necessarily care about us in any positive way.

Actually, the argument is rarely used to prove that any specific religion is right.  It merely establishes the existence of an infinite being--whether he actually cares about us is a matter of historical record.

I'm not quite getting where you derive this idea about 'obsolete metaphysical assumptions'.  What do you understand to be the primitive notion of cause and effect, and how is that any different than what we believe today?  I'll pose the same question with regard to an infinite regress and the mind.

The 'cause' of some event can often be an indefinitely large number of contributing factors, with some 'proximate' relatively trivial event triggering a far larger event, like the final snowflake landing on an unstable pile-up of snow on a mountain slope. The 'thinking' behind ideas like the 'principle of sufficient reason' is just so obsolete and out of touch with the modern understanding. Modern Complexity Theory has gone utterly beyond any of those ideas, the implications what can emerge from sufficiently complex deterministic systems.

Aquinas's vision of causality included the idea causes involve more than just a one to one correspondence between two things.  For example, he would have acknowledged that fire, in addition to the lighter, is also caused by the state of oxygen, gravity, and many other mitigating factors.  So, I do not see how your objection is relevant.  Do you have a specific philosopher in mind, or are you just putting them all under an umbrella based on a select few that you've read?

The principle of sufficient reason is not the same as the law of causality.  Beyond the argument from virtual particles, which itself is subject to debate, I do not see where you are coming from.  How is it false?   

Quote:
Then there is the simplistic understanding of 'determinism' and what they thought it implied.

The word 'determinism' is relatively new.  I can't think of any classic philosophers who mentioned it, although the issue of free will was obviously prevalent.   

Quote:
The idea of chaos due to feedback in non-linear systems, leading to situations popularly characterised as the 'Butterfly Effect' are related, but add a dimension where a difference in magnitude or direction of some event that can literally approach zero may determine which of two or more trajectories some fully deterministic process may take that can lead to dramatically different large-scale outcomes. This means that fully deterministic systems can behave in an absolutely unpredictable way. 

Determinism has nothing to do with predictability.  Is everything determined, or is it not? 

Quote:
Then there was the old idea that all 'motion' required a continuing 'mover', whereas only change of motion requires a cause.

How are you understanding 'motion' here?  This is an important question because the way Aristotle used to the term had nothing to do with the way that Newton used it.

Quote:
And motion is in fact 'energy', which is a fundamental of existence, only changing its form, so 'motion' is always with us, and absolutely does not need a 'mover'.

Again, how are you understanding 'motion'?

Quote:
Then you throw in Quantum Mechanics, which makes randomness and uncertainty fundamentals of reality, and then there is Einstein's Relativity, of course.

Uncertainty is nothing new.  In fact, Aristotle denied that the principle of bivalence applied categorically, and Jan Lukasiewicz formalized multi-valued systems without knowledge of quantum mechanics.

Quote:
And the failure to grasp that matter is in no way 'mere matter', it is a higher form of 'condensed' energy, absolutely required for any persistent structure needed in turn to support complex processes, such as living organisms and cognition.

What does that have to do with the law of causality, principle of sufficient reason, or anything else that you mentioned?

Quote:
They had naive ideas on 'infinity' and infinite summation ( Zeno's 'paradoxes' ).

Infinite regress only leads to any real infinities if it is assumed the 'causes' are consistently greater in some sense that their effect. 

All causes are qualitatively greater than their effects, unless you mean something different by 'cause'.  This is an issue of logic, not of science.  Causality involves agents, by virtue of their degree of perfection, producing something else--and your only alternative here is that something gives to itself what it does not have, which is nonsense. 

And even if that was not true, the notion of an infinite quantity of real things is still absurd for reasons irrelevant to the nature of causality, e.g., the cardinality of the set of all even numbers would be equal to that of all even and odd numbers put together--even though the cardinalilty of the former should be double to that of the latter.

Quote:
The sum of an infinite number of finite quantities, such as 1 + 1/2 + 1/4 + 1/8 +... = 2.0, ie, quite finite.

Mathematics does not magically map itself onto reality.  In reality, there are no perfect circles, infinite sets of natural numbers, or Pythagorean theorems.  These are abstract notions with no real existence.  That the sum of an abstract infinite series converges has nothing to do with the behaviors of real things and the implication of causality for those things.  Sorry, this just does not work.

Quote:
The picture we have now of the nature of mind and its dependence on the brain, emerging from many, many studies presents a far different picture from what was assumed then.

It is not as clear cut as you are presenting it.  The bridge principles between mental states and physical states have been shaken up by discoveries in biology involving the realizability of similar mental states in different species.  To counteract this, reductionists will make a disjunctive move, which itself is controversial.  So, we do not know absolutely that the mind depends on the brain, and many interpret the evidence as suggesting that the mind can be realized through many different mediums--and why should a disembodied mind be excluded from that?

Quote:
The very idea that intuition and introspection and 'pure' reason were all that were needed to grasp 'truth' is a basic fallacy underlying much of this obsolete thinking, the often expressed contempt for 'empiricism', which persists even today in many sub-cultures.

What science is this? 

Quote:
The lack of understanding of all these ideas means their speculations are worthless, their arguments void.

IOW, they were wrong on just about everything that mattered.

You do not know that they were wrong; you believe they were wrong. 


BobSpence
High Level DonorRational VIP!ScientistWebsite Admin
BobSpence's picture
Posts: 5939
Joined: 2006-02-14
User is offlineOffline
 The Principle of

 The Principle of Sufficeient Reason is no so much wrong, as empty of significance or utility, since what would be 'sufficient' to cause or determine some fact or event is not defined in it, and since the ultimate 'reason' foe any event or existence can of any scale down to the Planck scale, or perhaps nothing more than the underlying Heisenberg Uncertainty.

The Law of causality is not actually a Law, just a definition of 'cause' and 'effect'. 

Determinism has everything to do with predictability, it means entirely caused by prior events and states. Which logically implies predictability, in principle, and that given any total state of reality at soem point, only one course of events can follow.

Chaos theory has shown that even with 'perfect' knowledge of state of existence, the outcome will not necessarily be predictable, ie follow exactly the same path.

What has been added to Determinism in the naturalistic/scientific understanding, is Quantum Uncertainty which is not simply lack of complete knowledge of the state of reality, but states that it is categorically impossible to know or define this state with precision finer than the limits defined by Heisenberg.

Motion embodies energy. Ordinary physical motion through space carries with it a definable amount of energy, e = 1/2 X mass X velocity2. More metaphorical applications of 'motion' mean change of state, not position. And any such change typically requires an input of energy, all within the constraint of the Laws of Thermodynamics.

Causes are NOT necessarily 'greater' than there effects, in any sense, it does not follow from logic. It logically cannot be so, since a fundamental aspect of interactions is feedback, where event or state A causes B, which in turn causes or effects A, in a positive or negative sense, to any degree from small to entirely.

The finite mathematical sum of convergent infinite series means that any sequence of cause and effect really does mean that as long as the duration of each prior event in a sequence is definitely smaller than what follows, the conclusion MUST apply. Mathematics presents idealized, simplified models of reality, but reality cannot violate mathematical truths when they accurately describe some aspect of it. Two clearly stable and identifiable objects plus another two will always give you four such objects.

Finding correspondences in behaviour and cognition, as far as it can be determined by cleverly designed tests, in other creatures, massively reinforces the theories of cognition, and its evolutionary origins.

The idea that truth about reality could be attained by 'pure' reason was a a fundamental belief from the time of the Greeks, and earlier, onward, and lead to repeated errors, such as the idea that heavier objects fall faster then lighter objects, and that the realm beyond the Moon was a different order of reality.

You also only 'believe' you are right. I, on the other hand, can refer to massive amounts of hard evidence to support my position.

I am genuinely grateful to you for confirming that Metaphysics is a pile of dog crap, producing mostly error rather than actual knowledge and understanding.

EDIT:
Philosophers used to define "knowledge" as "justified true belief", and some still do, and that is an utterly stupid definition. Plato was a fool. Aristotle not quite so bad but still pretty silly, even by the standards of the day. Aquinas seems to have been a bit better, but still was stuck with a lot of erroneous metaphysical nonsense. None of those guys deserve to be treated as authorities on anything. That doesn't mean that they didn't occsasionally say something with some truth or insight, but there was so little understanding of "Life, the Universe, and Everything" compared to what is known with high degrees of confidence today, that their arguments are now of little more than historical significance.

Do you also happen to believe in the illogical proposition known as 'God'?

If there is any other way I can belittle and insult your beliefs, please let me know.

Favorite oxymorons: Gospel Truth, Rational Supernaturalist, Business Ethics, Christian Morality

"Theology is now little more than a branch of human ignorance. Indeed, it is ignorance with wings." - Sam Harris

The path to Truth lies via careful study of reality, not the dreams of our fallible minds - me

From the sublime to the ridiculous: Science -> Philosophy -> Theology


Mr_Metaphysics (not verified)
Posts: 4294964976
Joined: 1969-12-31
User is offlineOffline
BobSpence1 wrote: The

BobSpence1 wrote:

 The Principle of Sufficeient Reason is no so much wrong, as empty of significance or utility, since what would be 'sufficient' to cause or determine some fact or event is not defined in it, and since the ultimate 'reason' foe any event or existence can of any scale down to the Planck scale, or perhaps nothing more than the underlying Heisenberg Uncertainty.

Your first statement here is that the PSR is empty of significance--'significant' meaning 'important'.  So you are claiming here that the PSR is not important, to which I will claim that its relative importance has nothing to do with its veracity.  And provided that it is veracious, you're obliged to accept a key premise in the cosmological argument; namely, anything that exists has a sufficient reason for its existence--either in itself or in something else.

Your second statement is that the PSR is empty of utility--'utility' meaning 'usefulness'.  That's subjective; obviously it is not useful to you because you have no desire to propound any argument which relies upon it.  However, it is useful for the proponent of the cosmological argument, and once again you've made no claim regarding its veracity except to say that it is 'not so much wrong' (which I take to meant hat you accept the key premise in the cosmological argument).

That sufficient reason is ultimately reducible to the Planck scale does not address anything pertinent to the discussion.  Again, you've seemingly accepted that it is true--now you have to deal with the argument that utilizes it.

Quote:
The Law of causality is not actually a Law, just a definition of 'cause' and 'effect'.

If it is true in all places at all times, then it has earned a right to the title.

Quote:
Determinism has everything to do with predictability

It does not.  Just because an event is determined does not mean that it is predictable, and just because it is predictable does not mean that it is indeterminate.  We often times predict things that we normally conceive to be indeterminate such as human choices, and we often have a difficult time predicting things--such as rain storms--that are determined.  (Count the times meteorologists have been wrong!)

Quote:
What has been added to Determinism in the naturalistic/scientific understanding, is Quantum Uncertainty which is not simply lack of complete knowledge of the state of reality, but states that it is categorically impossible to know or define this state with precision finer than the limits defined by Heisenberg.

That's only relative to the observer.  The reason that it is scientifically impossible for us to predict, say, the movement of electrons is that our epistemic confidence in their behavior, as with any other physical object, trades upon our measurements; however, our measurements are what's affecting the electron, because our measurements involve the application of energy sources that inevitably affect it.  This does not at all mean that quantum mechanics is inherently indeterminate, or that the principle of sufficient reason is violated.  If we can ever find a different way to measure things at the Planck scale, then perhaps it will no longer spuriously appear to be a cauldron of lawlessness.

Quote:
Motion embodies energy. Ordinary physical motion through space carries with it a definable amount of energy, e = 1/2 X mass X velocity2. More metaphorical applications of 'motion' mean change of state, not position. And any such change typically requires an input of energy, all within the constraint of the Laws of Thermodynamics.

I'm not sure what you are disputing here, as the Thomistic idea of motion is completely in line with Newtonian mechanics.

Quote:
Causes are NOT necessarily 'greater' than there effects, in any sense, it does not follow from logic.

It absolutely does follow from logic.  Causality is the production of an effect, which involves an efficient cause conferring an inherent finite surplus of perfection onto something else.  Therefore, the efficient cause is, by logical necessity, qualitatively greater than whatever effect it instantiates, because the efficient cause necessarily has a higher surplus of perfection than the effect.  This is an a priori proposition.  Now your confusion may be with the idea that causality involves some one-to-one correspondence, but that has nothing to do with it.  Causality involves the action of many different substances; e.g., the flame isn't necessarily less great than the lighter, but it is less great than the combination of the lighter, air, gravity, and so on.

Quote:
The finite mathematical sum of convergent infinite series means that any sequence of cause and effect really does mean that as long as the duration of each prior event in a sequence is definitely smaller than what follows, the conclusion MUST apply. Mathematics presents idealized, simplified models of reality, but reality cannot violate mathematical truths when they accurately describe some aspect of it. Two clearly stable and identifiable objects plus another two will always give you four such objects.

Just declaring that mathematics somehow confers itself upon reality does not make it so.  Again, in the world of mathematics, there are perfect circles, triangles, and natural numbers; can you actually locate a perfect circle, or the number '7'?  If so, where are they?  You cannot look inside a physical object and find any natural number.  The only ontology that mathematics has in the real world is a series of ink marks on a piece of paper; other than that, they do not exist.  In a world where no minds exist, there is no '1 + 1 = 2'.  

Quote:
Finding correspondences in behaviour and cognition, as far as it can be determined by cleverly designed tests, in other creatures, massively reinforces the theories of cognition, and its evolutionary origins.

Did you not understand what I said?  Are you familiar with multiple realizability theory, reductionism, the disjunctive move, and the explanatory response?

Quote:
The idea that truth about reality could be attained by 'pure' reason was a a fundamental belief from the time of the Greeks, and earlier, onward, and lead to repeated errors, such as the idea that heavier objects fall faster then lighter objects, and that the realm beyond the Moon was a different order of reality.

Now you are just being ridiculous.  Aristotle was the authority on natural science, and he was not a Platonist.  I'm not even sure that he was a rationalist, since his theory of knowledge specifically traded upon the reliability of perception.  

And your statement is self-refuting; what empirical reason do you have to believe that empirical reasoning produces fundamental truths about reality?  If you say that it is your empirical observations, then you are begging the question because that would require the prior assumption that empirical observation produces such truths.  If you say that it is pure reasoning, then you've repudiated the position you've just espoused.

Quote:
You also only 'believe' you are right. I, on the other hand, can refer to massive amounts of hard evidence to support my position.

You haven't produced a shred of evidence for your position--you've only given me diatribes and speculation.

 

 


BobSpence
High Level DonorRational VIP!ScientistWebsite Admin
BobSpence's picture
Posts: 5939
Joined: 2006-02-14
User is offlineOffline
 The PSR says nothing -

 The PSR says nothing - asserting that everything has a 'sufficient' reason for its existence without providing any constraints or definitions on what qualifies as 'sufficient', is a totally vacuous, empty, pointless assertion.

The Law of Cause and Effect, as a defintion, obviously applies everywhere, bit tells us nothing about reality, as to where cause and effect apply.

If you have made a serious, meaningful response to my question, you are out of your tiny brain.

Actually, due to Chaotic effects, a deterministic process may indeed not be predictable, that would apply to the weather, of course, But only a strictly deterministic process is fully predictable.

Energy applies to more than just motion of objects according to Newtonian mechanics, and is all that is required to pre-exist before any action can occur.

"conferring an inherent finite surplus of perfection onto something" ?? Are you kidding?? Where did you get that stuff from?? That is utterly meaningless crap. Unless you can provide the logical justification for describing 'cause' in those terms. Ahh I see you describe it as an "a priori proposition". IOW you have no logical justification for it. It is an idea you pulled out of your rear aperture.

I don't believe causality necessarily involves a 'one-to-one' correspondence, it involves an accumulation of influences, such as temperature, pressure, air movement, etc, etc, which eventually combine to trigger some event, such as pushing an object over some threshhold, so it can roll down a slope. If the object was finely balanced the total 'cause' need  only be very small. Actually comparing cause and effect as to 'greatness' doesn't often make sense. 

In terms of  analysing infinite regress situations, as long as the total energy and duration involved in each notional cause-effect sequence, the total sequence back to an infinitesimal origin, or in reality, something equivalent to a quantum twitch, does not involve any physical infinities of time or energy, therefore is not problematic.

Whether or not minds exist, reality cannot defy logic, since logic is based on the existence of a basic coherence to existence - our formulation of logic is derived from the most fundamental observations of reality.

Your comments do not, and cannot, contradict the conclusions drawn from the observation of the behavior of various creatures including ourselves. They are just pointless philosophical pedantry. I am arguing from the PoV of scientific investigation.

Whatever the imperfections of empiricism, they have infinitely more grounding than 'a priori' intuitions and assumptions.

 

Favorite oxymorons: Gospel Truth, Rational Supernaturalist, Business Ethics, Christian Morality

"Theology is now little more than a branch of human ignorance. Indeed, it is ignorance with wings." - Sam Harris

The path to Truth lies via careful study of reality, not the dreams of our fallible minds - me

From the sublime to the ridiculous: Science -> Philosophy -> Theology


Mr_Metaphysics (not verified)
Posts: 4294964976
Joined: 1969-12-31
User is offlineOffline
BobSpence1 wrote: The PSR

BobSpence1 wrote:

 The PSR says nothing - asserting that everything has a 'sufficient' reason for its existence without providing any constraints or definitions on what qualifies as 'sufficient', is a totally vacuous, empty, pointless assertion.

If you woke up in the morning to find that a fire was blazing in your kitchen, would you assume that it was there for absolutely no reason at all?  That the fire was just a matter of brute fact?  Or would you attempt to figure out what caused the fire so as to prevent it from happening in the future?

Quote:
If you have made a serious, meaningful response to my question, you are out of your tiny brain.

Now you are just attacking me.  Are you capable of rational discussion?  This is the philosophy forum; if you hate philosophy, then you shouldn't post here.

Quote:
Your comments do not, and cannot, contradict the conclusions drawn from the observation of the behavior of various creatures including ourselves. They are just pointless philosophical pedantry. I am arguing from the PoV of scientific investigation.

You are not even a scientist; you are a computer programmer.  You do not speak for scientists.  I'm guessing that you are only restricting your reading of scientists to guys like Victor Stenger and Richard Dawkins.


Ktulu
atheist
Posts: 1831
Joined: 2010-12-21
User is offlineOffline
Mr_Metaphysics wrote:Hi, I'm

Mr_Metaphysics wrote:

Hi, I'm new to the forum.  I am a theist, and I hope we can be civil in our discourse. 

You need to demonstrate how it is possible for there to be an endless sequence of anything, let alone Big Bangs.  I do not see how the idea of an actual infinite quantity is even logically coherent.

'First cause' is not a benign notion; many things necessarily follow from the idea, such as eternity, timelessness, self-existence, and immateriality.

Hello, and welcome to the forum.

1. You ask for a demonstration of an endless sequence, due to the fact that an infinite quantity is illogical. 

2. You claim that first cause is a malignant notion (not benign), and hence, eternity and timelessness?

I'm going to keep reading this as I see there are is much more to it, but as of right now I'm confused..

"Don't seek these laws to understand. Only the mad can comprehend..." -- George Cosbuc


Ktulu
atheist
Posts: 1831
Joined: 2010-12-21
User is offlineOffline
Mr_Metaphysics

Mr_Metaphysics wrote:

Actually, the argument is rarely used to prove that any specific religion is right.  It merely establishes the existence of an infinite being--whether he actually cares about us is a matter of historical record.

You cannot prove any religion, specific or otherwise, is 'right' otherwise it wouldn't be a religion.  The argument is used to justify a 'philosophical' ground for a belief.  And I'm assuming that by the term 'infinite being' you imply that it has feelings or intelligence.  Why not an infinite hammer that smashes atoms into new universes all the time? Why not a pendulum that just swings every 50 billion years or so for a new big bang? Why not an eternal chicken? Why is God the only option out there? I mean they're all just as logically possible, and if you think about it, an eternal chicken raises less questions then an eternal being with 'omni-' everything.  

Why is it that when it comes to reasoning God you apply obscure philosophical concepts, but when it comes to reasoning whether he cares you apply obscure historical 'evidence' from the book? It seems a bit like picking and choosing here.

 

 

"Don't seek these laws to understand. Only the mad can comprehend..." -- George Cosbuc


Mr_Metaphysics (not verified)
Posts: 4294964976
Joined: 1969-12-31
User is offlineOffline
Ktulu wrote:Mr_Metaphysics

Ktulu wrote:

Mr_Metaphysics wrote:

Hi, I'm new to the forum.  I am a theist, and I hope we can be civil in our discourse. 

You need to demonstrate how it is possible for there to be an endless sequence of anything, let alone Big Bangs.  I do not see how the idea of an actual infinite quantity is even logically coherent.

'First cause' is not a benign notion; many things necessarily follow from the idea, such as eternity, timelessness, self-existence, and immateriality.

Hello, and welcome to the forum.

1. You ask for a demonstration of an endless sequence, due to the fact that an infinite quantity is illogical. 

2. You claim that first cause is a malignant notion (not benign), and hence, eternity and timelessness?

I'm going to keep reading this as I see there are is much more to it, but as of right now I'm confused..

It's a fair question.  I thought 'benign' referred to something that was inactive, and I thought I was using it to refer to an idea through which no further implications could be made.  I chatted with someone else once who used the term in that manner, but I never bothered to look it up.  I just looked it up and it does not mean what I thought it meant.  Please forgive my misuse of the term.


Mr_Metaphysics (not verified)
Posts: 4294964976
Joined: 1969-12-31
User is offlineOffline
Ktulu wrote:The argument is

Ktulu wrote:

The argument is used to justify a 'philosophical' ground for a belief.  And I'm assuming that by the term 'infinite being' you imply that it has feelings or intelligence. Why not an infinite hammer that smashes atoms into new universes all the time? Why not a pendulum that just swings every 50 billion years or so for a new big bang? Why not an eternal chicken? Why is God the only option out there? I mean they're all just as logically possible, and if you think about it, an eternal chicken raises less questions then an eternal being with 'omni-' everything.  

'Infinite' in this sense is not a quantitative notion; it is a qualitative notion referring to unlimitedness.  Since a hammer is a material object and materiality is a limitation, 'infinite hammer' is therefore a contradiction in terms.

Quote:
Why is it that when it comes to reasoning God you apply obscure philosophical concepts, but when it comes to reasoning whether he cares you apply obscure historical 'evidence' from the book? It seems a bit like picking and choosing here.

I do not find them to be obscure.  This view was shared by the early Church fathers and the theistic philosophers of the Renaissance.  Even today, I think the average theist off the street would say that 'God' is the all-powerful all-knowing creator of the universe.  

The issue with his personal relationship to humanity is an empirical question; hence, we cannot prove it just by appealing to the idea.


BobSpence
High Level DonorRational VIP!ScientistWebsite Admin
BobSpence's picture
Posts: 5939
Joined: 2006-02-14
User is offlineOffline
Mr_Metaphysics

Mr_Metaphysics wrote:

BobSpence1 wrote:

 The PSR says nothing - asserting that everything has a 'sufficient' reason for its existence without providing any constraints or definitions on what qualifies as 'sufficient', is a totally vacuous, empty, pointless assertion.

If you woke up in the morning to find that a fire was blazing in your kitchen, would you assume that it was there for absolutely no reason at all?  That the fire was just a matter of brute fact?  Or would you attempt to figure out what caused the fire so as to prevent it from happening in the future?

I would base any assumptions on my knowledge of the sorts of things that could cause a fire under those circumstances.

If I heard a click in a geiger counter, I would know that it was probably caused by a decaying atom, which in turn simply happened at random with a known probability. Somethings happen due to one or more pre-existing states of the local environment, plus some proximate triggering event, some don't.

The PSR simply encapsulates a common intuition. It does not deserve to be regarded as a 'principle'.

Quote:

Quote:
If you have made a serious, meaningful response to my question, you are out of your tiny brain.

Now you are just attacking me.  Are you capable of rational discussion?  This is the philosophy forum; if you hate philosophy, then you shouldn't post here.

Quote:
Your comments do not, and cannot, contradict the conclusions drawn from the observation of the behavior of various creatures including ourselves. They are just pointless philosophical pedantry. I am arguing from the PoV of scientific investigation.

You are not even a scientist; you are a computer programmer.  You do not speak for scientists.  I'm guessing that you are only restricting your reading of scientists to guys like Victor Stenger and Richard Dawkins.

I am a University qualified Electrical Engineer with an honors degree. This included Physics, Thermodynamics, Chemistry, Information Theory, an introduction to Einstein's Relativity, Control Systems including feed-back theory, etc.

Informally, I have been reading science magazines for 50 years, and now following the podcasts from Nature, Scientific American, AAAS, other science 'casts from the BBC, NOVA, NPR, PRI, among others.

I am more qualified to 'speak for scientists than a philosophy student.

This forum is also for Psychology, and I am fascinated with the psychology of those who have very different world-views from mine.

Favorite oxymorons: Gospel Truth, Rational Supernaturalist, Business Ethics, Christian Morality

"Theology is now little more than a branch of human ignorance. Indeed, it is ignorance with wings." - Sam Harris

The path to Truth lies via careful study of reality, not the dreams of our fallible minds - me

From the sublime to the ridiculous: Science -> Philosophy -> Theology


BobSpence
High Level DonorRational VIP!ScientistWebsite Admin
BobSpence's picture
Posts: 5939
Joined: 2006-02-14
User is offlineOffline
Immateriality is a much more

Immateriality is a much more severe limitation on possibilities than "materiality".

You are just perpetuating an ancient intuition.

 

Favorite oxymorons: Gospel Truth, Rational Supernaturalist, Business Ethics, Christian Morality

"Theology is now little more than a branch of human ignorance. Indeed, it is ignorance with wings." - Sam Harris

The path to Truth lies via careful study of reality, not the dreams of our fallible minds - me

From the sublime to the ridiculous: Science -> Philosophy -> Theology


Mr_Metaphysics (not verified)
Posts: 4294964976
Joined: 1969-12-31
User is offlineOffline
BobSpence1

BobSpence1 wrote:

Mr_Metaphysics wrote:

BobSpence1 wrote:

 The PSR says nothing - asserting that everything has a 'sufficient' reason for its existence without providing any constraints or definitions on what qualifies as 'sufficient', is a totally vacuous, empty, pointless assertion.

If you woke up in the morning to find that a fire was blazing in your kitchen, would you assume that it was there for absolutely no reason at all?  That the fire was just a matter of brute fact?  Or would you attempt to figure out what caused the fire so as to prevent it from happening in the future?

I would base any assumptions on my knowledge of the sorts of things that could cause a fire under those circumstances.

If I heard a click in a geiger counter, I would know that it was probably caused by a decaying atom, which in turn simply happened at random with a known probability. Somethings happen due to one or more pre-existing states of the local environment, plus some proximate triggering event, some don't.

The PSR simply encapsulates a common intuition. It does not deserve to be regarded as a 'principle'.

It's true in all places at all times; therefore it's a principle.

Quote:
I am a University qualified Electrical Engineer with an honors degree. This included Physics, Thermodynamics, Chemistry, Information Theory, an introduction to Einstein's Relativity, Control Systems including feed-back theory, etc.

Informally, I have been reading science magazines for 50 years, and now following the podcasts from Nature, Scientific American, AAAS, other science 'casts from the BBC, NOVA, NPR, PRI, among others.

I am more qualified to 'speak for scientists than a philosophy student.

I'm not a philosophy student; I graduated more than 5 years ago.  At this stage, I am an enthusiast.  The fact is that you have no idea what kind of research that I've done, and during our conversations you've offered nothing of substance with regard to scientific facts.  You simply repeat lines by Dawkins or Victor Stenger and act like this is somehow first class scholarship on your part.

The bottom line is that you have no arguments, so now you are only resorting to character attacks--focusing a good portion of your posts to attacking my education or credentials.  I do not need to have a degree in science in order to know that any of this is consistent with my theology.

There does not seem to be a single thread on this forum discussing psychology.  You are here engaging in philosophical discussions, while simultaneously attacking philosophy.  From what I can gather, your readings of philosophy are cursory, and you are unaware of advancements going on in philosophical circles today regarding formal systems, mind-body theories, epistemology, philosophical ethics, and philosophy of science.  Again, this is a philosophy forum (the fact that it includes 'psychology' seems merely inconsequential at this point, and the subject lines have nothing to do with my personal psychology).  Why are you here?


Ktulu
atheist
Posts: 1831
Joined: 2010-12-21
User is offlineOffline
Mr_Metaphysics wrote:Ktulu

Mr_Metaphysics wrote:

Ktulu wrote:

The argument is used to justify a 'philosophical' ground for a belief.  And I'm assuming that by the term 'infinite being' you imply that it has feelings or intelligence. Why not an infinite hammer that smashes atoms into new universes all the time? Why not a pendulum that just swings every 50 billion years or so for a new big bang? Why not an eternal chicken? Why is God the only option out there? I mean they're all just as logically possible, and if you think about it, an eternal chicken raises less questions then an eternal being with 'omni-' everything.  

'Infinite' in this sense is not a quantitative notion; it is a qualitative notion referring to unlimitedness.  Since a hammer is a material object and materiality is a limitation, 'infinite hammer' is therefore a contradiction in terms.

I think this is the part I do not understand, bear with me here.  I was also at this point in my progression to becoming an atheist.  I understood to a certain degree the problem that a cosmological argument presents.  The thing that I don't understand is this jaunt in logic.  Right here, you present your case for a cosmological argument, which we all agree with in concept, then you assume 'ergo God'.  I mean... I understand that the hammer is a material object, think of it's immaterial equivalent.  It was a poor example of an alternative to a conscious being.  The answer doesn't have to be as simple as arriving from nothing ( read nothing as quantum foam ), but anything more then that is a non sequitur.  Explain this portion of your argument again please.  

Let me reiterate where we are with my question.  Causation implies that 'SOMETHING' created the Big Bang ( or whatever equivalent of universe starting point you believe in, it's not relevant at this point ).  What is the next step in assuming that this 'SOMETHING' is God, or even sentient?

Mr_Metaphysics wrote:

Quote:
Why is it that when it comes to reasoning God you apply obscure philosophical concepts, but when it comes to reasoning whether he cares you apply obscure historical 'evidence' from the book? It seems a bit like picking and choosing here.

I do not find them to be obscure.  This view was shared by the early Church fathers and the theistic philosophers of the Renaissance.  Even today, I think the average theist off the street would say that 'God' is the all-powerful all-knowing creator of the universe.  

The issue with his personal relationship to humanity is an empirical question; hence, we cannot prove it just by appealing to the idea.

But the two approaches almost seem to contradict, on one hand you claim that god is outside nature and cannot be empirically detected, and on the other hand you present what you consider empirical historical evidence for this said creator.  If you arrive to your understanding of God the way you have (philosophy, metaphysically) , then how does the Bible follow from that?  That's an even bigger non sequitur then the previous one in my opinion. 

"Don't seek these laws to understand. Only the mad can comprehend..." -- George Cosbuc


BobSpence
High Level DonorRational VIP!ScientistWebsite Admin
BobSpence's picture
Posts: 5939
Joined: 2006-02-14
User is offlineOffline
It is entirely appropriate

It is entirely appropriate to discuss problems with philosophy on a philosophy forum.

A square has 4 sides is also true everywhere, as is the assertion that 'up' is the opposite of 'down'. Those statements have more utility than that Things can be caused.

You are apparently using infinite incorrectly. You appear to mean 'unbounded' or indefinitely large, which are not the same thing at all.

EDIT: Pointing your clear logical fallacies and naked assertions/assumptions (AKA 'a priori' ) is 'just insulting you'?

Favorite oxymorons: Gospel Truth, Rational Supernaturalist, Business Ethics, Christian Morality

"Theology is now little more than a branch of human ignorance. Indeed, it is ignorance with wings." - Sam Harris

The path to Truth lies via careful study of reality, not the dreams of our fallible minds - me

From the sublime to the ridiculous: Science -> Philosophy -> Theology


Mr_Metaphysics (not verified)
Posts: 4294964976
Joined: 1969-12-31
User is offlineOffline
Ktulu wrote:Let me reiterate

Ktulu wrote:

Let me reiterate where we are with my question.  Causation implies that 'SOMETHING' created the Big Bang ( or whatever equivalent of universe starting point you believe in, it's not relevant at this point ).  What is the next step in assuming that this 'SOMETHING' is God, or even sentient?

All causes are greater than their effects; therefore, the First Cause would be qualitatively greater than everything that is--as such, it possesses great making properties such as omniscience, omnipotence, omnipresence, eternity, self-existence, immateriality, and so forth.  You don't have to call this being 'God', but it is worshipful nonetheless.

Quote:
But the two approaches almost seem to contradict, on one hand you claim that god is outside nature and cannot be empirically detected, and on the other hand you present what you consider empirical historical evidence for this said creator.

We can't actually perceive God with our senses, but we can perceive the results of his actions--including the Bible and miracles.

Quote:
If you arrive to your understanding of God the way you have (philosophy, metaphysically) , then how does the Bible follow from that?  That's an even bigger non sequitur then the previous one in my opinion. 

The Bible does not follow from that.  Most of the arguments only produce, as they say, the 'God of the Philosophers'--an impersonal, deistic God.

The Bible is in a separate domain of evidence.  St. Thomas Aquinas made a distinction between preambles and mysteries.  Some reasons for belief are evident in nature or logic, but others are only evident via special revelation.  No logical argument can prove that God is Triune, or that God loves us.


mellestad
Moderator
Posts: 2929
Joined: 2009-08-19
User is offlineOffline
Mr_Metaphysics wrote:Ktulu

Mr_Metaphysics wrote:

Ktulu wrote:

Let me reiterate where we are with my question.  Causation implies that 'SOMETHING' created the Big Bang ( or whatever equivalent of universe starting point you believe in, it's not relevant at this point ).  What is the next step in assuming that this 'SOMETHING' is God, or even sentient?

All causes are greater than their effects; therefore, the First Cause would be qualitatively greater than everything that is--as such, it possesses great making properties such as omniscience, omnipotence, omnipresence, eternity, self-existence, immateriality, and so forth.  You don't have to call this being 'God', but it is worshipful nonetheless.

 

They are?  I assume you mean this in some philosophic sense, rather than a material sense, right?  In a physical sense that is obviously not the case.

Everything makes more sense now that I've stopped believing.


Mr_Metaphysics (not verified)
Posts: 4294964976
Joined: 1969-12-31
User is offlineOffline
BobSpence1 wrote:It is

BobSpence1 wrote:

It is entirely appropriate to discuss problems with philosophy on a philosophy forum.

This forum was created specifically for philosophy enthusiasts.  It has the following tag line: "Aesthetics, Epistemology, Ethics, Logic, Metaphysics, and all other Philosophical discussion. Additional Psychology discussion!" It is for people who wish to engage in philosophical discussion--you are not one of those people.  

Quote:
You are apparently using infinite incorrectly. You appear to mean 'unbounded' or indefinitely large, which are not the same thing at all.

Different words mean different things depending on context.  In chemistry, the word 'substance' means something entirely different than when it is used in metaphysics.  Likewise, the word 'cell' means something different in the context of law enforcement than it does in the context of biology.  That doesn't make one usage 'incorrect', it just means that the terms are equivocal.

'Infinite' in metaphysics refers to qualitative infinitude, 'infinite' in mathematics refers to quantitative infinitude. 

Quote:
EDIT: Pointing your clear logical fallacies and naked assertions/assumptions (AKA 'a priori' ) is 'just insulting you'?

You aren't just pointing out fallacies; you are calling me stupid, attacking my education, and questioning my integrity--all because I have a different belief than you.  It's borderline fascism, and I'm glad that people like you are not running the world.  


Mr_Metaphysics (not verified)
Posts: 4294964976
Joined: 1969-12-31
User is offlineOffline
mellestad

mellestad wrote:

Mr_Metaphysics wrote:

Ktulu wrote:

Let me reiterate where we are with my question.  Causation implies that 'SOMETHING' created the Big Bang ( or whatever equivalent of universe starting point you believe in, it's not relevant at this point ).  What is the next step in assuming that this 'SOMETHING' is God, or even sentient?

All causes are greater than their effects; therefore, the First Cause would be qualitatively greater than everything that is--as such, it possesses great making properties such as omniscience, omnipotence, omnipresence, eternity, self-existence, immateriality, and so forth.  You don't have to call this being 'God', but it is worshipful nonetheless.

 

They are?  I assume you mean this in some philosophic sense, rather than a material sense, right?  In a physical sense that is obviously not the case.

The nature of causality is not something that is discussed in natural science.  Science merely posits models based on a reinterpretation of 'causality' which entails predictability according to law.  It says nothing about what cause and effect actually is, or the qualitative perfections of individual causes and effects.  It is only in philosophy where this is discussed.


cj
atheistRational VIP!
cj's picture
Posts: 3330
Joined: 2007-01-05
User is offlineOffline
mellestad wrote:

mellestad wrote:

Mr_Metaphysics wrote:

Ktulu wrote:

Let me reiterate where we are with my question.  Causation implies that 'SOMETHING' created the Big Bang ( or whatever equivalent of universe starting point you believe in, it's not relevant at this point ).  What is the next step in assuming that this 'SOMETHING' is God, or even sentient?

All causes are greater than their effects; therefore, the First Cause would be qualitatively greater than everything that is--as such, it possesses great making properties such as omniscience, omnipotence, omnipresence, eternity, self-existence, immateriality, and so forth.  You don't have to call this being 'God', but it is worshipful nonetheless.

They are?  I assume you mean this in some philosophic sense, rather than a material sense, right?  In a physical sense that is obviously not the case.

 

Obviously this dude is unfamiliar with the domino effect.  Watch near the ending, the dominoes are obviously falling with more force than they started with - from the cumulative effects of gravity, natch.  But it all starts with one small flick of a finger.

 

 

 

-- I feel so much better since I stopped trying to believe.

"We are entitled to our own opinions. We're not entitled to our own facts"- Al Franken

"If death isn't sweet oblivion, I will be severely disappointed" - Ruth M.


cj
atheistRational VIP!
cj's picture
Posts: 3330
Joined: 2007-01-05
User is offlineOffline
Mr_Metaphysics wrote:The

Mr_Metaphysics wrote:

The nature of causality is not something that is discussed in natural science.  Science merely posits models based on a reinterpretation of 'causality' which entails predictability according to law.  It says nothing about what cause and effect actually is, or the qualitative perfections of individual causes and effects.  It is only in philosophy where this is discussed.

 

The nature of causality - as in, what causes effects?  As in, the type of cause?  As in, how a cause is generated?  As in, why there are causes?

I don't know, seems to me a lot of scientists address the nature of causality very regularly.  Why do we have earthquakes, why do train lights look red as the train passes us, why do we have volcanoes, why do we have fossils of creatures that no one has ever seen alive, why is the climate changing, why, why, why, why.......

Qualitative perfections of individual causes and effects?  They are what they are - no perfection required or needed.

 

-- I feel so much better since I stopped trying to believe.

"We are entitled to our own opinions. We're not entitled to our own facts"- Al Franken

"If death isn't sweet oblivion, I will be severely disappointed" - Ruth M.


Mr_Metaphysics (not verified)
Posts: 4294964976
Joined: 1969-12-31
User is offlineOffline
cj wrote:Mr_Metaphysics

cj wrote:

Mr_Metaphysics wrote:

The nature of causality is not something that is discussed in natural science.  Science merely posits models based on a reinterpretation of 'causality' which entails predictability according to law.  It says nothing about what cause and effect actually is, or the qualitative perfections of individual causes and effects.  It is only in philosophy where this is discussed.

 

The nature of causality - as in, what causes effects?  As in, the type of cause?  As in, how a cause is generated?  As in, why there are causes?

No.  The nature of causality, as in 'what exactly is causality'?  In other words, is it mere temporal succession? Is it an inherent conceptual connection between two things?  Is it objective?  Is it subjective?  Are causes simultaneous with effects?  Is it a one to one correspondence?  etc. etc. etc.

Quote:
I don't know, seems to me a lot of scientists address the nature of causality very regularly.

No, they do not.  They observe physical events happening in succession, and they create a model based on their observations.  They do not care about the aforementioned issues; they are merely concerned with whether the physical behavior is repeatable and what we may infer about the universe based on that repeated physical behavior.  At the philosophical level of explanation, scientists have not addressed anything--and they shouldn't because admittedly they are not philosophers.  

If you disagree with me, then please produce some sort of peer reviewed publication regarding scientific observations of causation.  Is there a law of causation in science telling us precisely what causation is, whether causes are greater than effects, and so forth?  If so, who put it together?  As far as I can tell, scientists *presuppose* causality in order to go forward with their methodology--in other words, they leave the philosophizing to the philosophers.

Quote:
Why do we have earthquakes, why do train lights look red as the train passes us, why do we have volcanoes, why do we have fossils of creatures that no one has ever seen alive, why is the climate changing, why, why, why, why.......

This has nothing to do with metaphysical issues regarding causes and effects.

Quote:
Qualitative perfections of individual causes and effects?  They are what they are - no perfection required or needed.

That does not even make sense--I take it you did not understand what I meant.  

 


Mr_Metaphysics (not verified)
Posts: 4294964976
Joined: 1969-12-31
User is offlineOffline
cj wrote:Obviously this dude

cj wrote:

Obviously this dude is unfamiliar with the domino effect.  Watch near the ending, the dominoes are obviously falling with more force than they started with - from the cumulative effects of gravity, natch.  But it all starts with one small flick of a finger.

You are really confused.  The instantiation of every effect you are observing is grounded upon a multitude of physical factors, the least of which includes someone's finger.  The sum total of all those factors constitutes the efficient cause, which by logical necessity must be greater than its effects.  Your only alternative is to believe that something can give itself what it does not have.

 

 


Atheistextremist
atheist
Atheistextremist's picture
Posts: 5134
Joined: 2009-09-17
User is offlineOffline
So many words expended

 

Mr_Metaphysics wrote:

We can't actually perceive God with our senses, but we can perceive the results of his actions - including the Bible and miracles.

Some reasons for belief are evident in nature or logic, but others are only evident via special revelation. 

 

and this seems to be the underlying weight of the thing  - that the Bible is the result of the actions of a god, that miracles exist and are the result of a god. That there's actually 'special revelation', whatever that might be.

Odd, Mr Met, that you've run so far with the mantle of science over your shoulders only to come to this point.

Do you truly believe in the immaterial - that the immaterial underlies the material somehow as a sort of inherent truth, undetectable and yet somehow obvious to the Platonist brain?

You can imagine how this looks to us materialists.

Interesting thread Mr Met et al, anyway. Such a shame things always trudge back to the same weary impasse. There is a god but he can't be known unless we first believe... 

 

 

"Experiments are the only means of knowledge at our disposal. The rest is poetry, imagination." Max Planck


Atheistextremist
atheist
Atheistextremist's picture
Posts: 5134
Joined: 2009-09-17
User is offlineOffline
Given your postion on the importance of physical factors

 

Mr_Metaphysics wrote:

cj wrote:

Obviously this dude is unfamiliar with the domino effect.  Watch near the ending, the dominoes are obviously falling with more force than they started with - from the cumulative effects of gravity, natch.  But it all starts with one small flick of a finger.

You are really confused.  The instantiation of every effect you are observing is grounded upon a multitude of physical factors, the least of which includes someone's finger.  The sum total of all those factors constitutes the efficient cause, which by logical necessity must be greater than its effects.  Your only alternative is to believe that something can give itself what it does not have.

 

Could you explain how the immaterial or the power of intelligence is able to impact on the material universe? You seem to be arguing against your own case. How can an immaterial deity be the cause of any physical thing?

 

 

"Experiments are the only means of knowledge at our disposal. The rest is poetry, imagination." Max Planck


mellestad
Moderator
Posts: 2929
Joined: 2009-08-19
User is offlineOffline
Mr_Metaphysics wrote:cj

Mr_Metaphysics wrote:

cj wrote:

Mr_Metaphysics wrote:

The nature of causality is not something that is discussed in natural science.  Science merely posits models based on a reinterpretation of 'causality' which entails predictability according to law.  It says nothing about what cause and effect actually is, or the qualitative perfections of individual causes and effects.  It is only in philosophy where this is discussed.

 

The nature of causality - as in, what causes effects?  As in, the type of cause?  As in, how a cause is generated?  As in, why there are causes?

No.  The nature of causality, as in 'what exactly is causality'?  In other words, is it mere temporal succession? Is it an inherent conceptual connection between two things?  Is it objective?  Is it subjective?  Are causes simultaneous with effects?  Is it a one to one correspondence?  etc. etc. etc.

Quote:
I don't know, seems to me a lot of scientists address the nature of causality very regularly.

No, they do not.  They observe physical events happening in succession, and they create a model based on their observations.  They do not care about the aforementioned issues; they are merely concerned with whether the physical behavior is repeatable and what we may infer about the universe based on that repeated physical behavior.  At the philosophical level of explanation, scientists have not addressed anything--and they shouldn't because admittedly they are not philosophers.  

If you disagree with me, then please produce some sort of peer reviewed publication regarding scientific observations of causation.  Is there a law of causation in science telling us precisely what causation is, whether causes are greater than effects, and so forth?  If so, who put it together?  As far as I can tell, scientists *presuppose* causality in order to go forward with their methodology--in other words, they leave the philosophizing to the philosophers.

Quote:
Why do we have earthquakes, why do train lights look red as the train passes us, why do we have volcanoes, why do we have fossils of creatures that no one has ever seen alive, why is the climate changing, why, why, why, why.......

This has nothing to do with metaphysical issues regarding causes and effects.

Quote:
Qualitative perfections of individual causes and effects?  They are what they are - no perfection required or needed.

That does not even make sense--I take it you did not understand what I meant.  

 

 

Why would anyone even care about those questions 'metaphysically', how are they even relevant to our experience?  It seems like asking a non-sense question to me...what's the point?  What problem exists that these questions need to find an answer for?  What powers of explanation do non-materialistic answers provide?

Everything makes more sense now that I've stopped believing.


Ktulu
atheist
Posts: 1831
Joined: 2010-12-21
User is offlineOffline
Mr_Metaphysics wrote:Ktulu

Mr_Metaphysics wrote:

Ktulu wrote:

Let me reiterate where we are with my question.  Causation implies that 'SOMETHING' created the Big Bang ( or whatever equivalent of universe starting point you believe in, it's not relevant at this point ).  What is the next step in assuming that this 'SOMETHING' is God, or even sentient?

All causes are greater than their effects; therefore, the First Cause would be qualitatively greater than everything that is--as such, it possesses great making properties such as omniscience, omnipotence, omnipresence, eternity, self-existence, immateriality, and so forth.  You don't have to call this being 'God', but it is worshipful nonetheless.

How are all causes greater then their effects? That makes no sense to me.  I think this is where we are getting separated.  It's not intuitively so, I can let a rock drop from an overpass exerting little to no effort, it would hit a car and cause a chain reaction accident where the forces involved are exponentially greater then the force I exerted on the rock.  

The only way I can even see that comment making remote sense is if you assume that the universe is part of God.  In which case you imply that god used part of himself as raw material.  That would necessitate the cause to be greater then the effect.  Is that what you're going for? I'm genuinely curious I'm not trying to be an asshole.  I really am trying to understand this logic gap in either mine or your train of thought.

If god used available material to create the universe, god need not be infinite, or omni-'x'.  He just needs to be in the right 'place' at the right 'time' ( I understand that time and place are concepts that make no sense in this context but just use your imagination ).

This is how I imagine it...  Sci-fi time.  1 million years into the future the human race has evolved extreme intelligence.  We learn to manipulate black holes.  We create this stable black hole type entity that feeds mater from this universe into 'else-where' outside this universe.  We then have created a different universe, are we god? or omni-'x'? ( I know I'm making a lot of silly assumptions here but every one of those is more plausible then then biblical god ).

 

"Don't seek these laws to understand. Only the mad can comprehend..." -- George Cosbuc


BobSpence
High Level DonorRational VIP!ScientistWebsite Admin
BobSpence's picture
Posts: 5939
Joined: 2006-02-14
User is offlineOffline
Science understands

Science understands causality, metaphysics apparently does not, or invents its own definition of it.

The understanding that correlation does not prove causation is a fundamental principle in scientific analysis, a genuinely useful principle, unlike PSR.

Establishing REAL causality is a fundamental aspect of scientific investigation. Anyone making the assertion that science only deals with simple succession of events betrays an almost unbelievable ignorance of science.

As an example to disprove the idea of 'necessary' relative magnitude of cause and effect, the energy required to topple a tall, standing object on a relatively small base, by exerting a horizontal force close to the top of the object, is much smaller than the energy released by its fall. By any measure, such a 'cause' is much smaller than its effect - there are no other contributing 'causes' to be added in. This is pretty much what is happening in the dominoes example.

The energy released by an explosive is vastly more than that in the fuse or detonator.

The basic point is that stored energy can be released or controlled by actions vastly smaller in every respect than the energy release they are controlling. It is meaningless to say the stored energy is part of the cause of is own release.

Even a dynamic flow of energy can be controlled and re-directed by means involving a fraction of the energy in the flow. 

 

Favorite oxymorons: Gospel Truth, Rational Supernaturalist, Business Ethics, Christian Morality

"Theology is now little more than a branch of human ignorance. Indeed, it is ignorance with wings." - Sam Harris

The path to Truth lies via careful study of reality, not the dreams of our fallible minds - me

From the sublime to the ridiculous: Science -> Philosophy -> Theology


cj
atheistRational VIP!
cj's picture
Posts: 3330
Joined: 2007-01-05
User is offlineOffline
mellestad

mellestad wrote:

Mr_Metaphysics wrote:

cj wrote:

Mr_Metaphysics wrote:

The nature of causality is not something that is discussed in natural science.  Science merely posits models based on a reinterpretation of 'causality' which entails predictability according to law.  It says nothing about what cause and effect actually is, or the qualitative perfections of individual causes and effects.  It is only in philosophy where this is discussed.

 

The nature of causality - as in, what causes effects?  As in, the type of cause?  As in, how a cause is generated?  As in, why there are causes?

No.  The nature of causality, as in 'what exactly is causality'?  In other words, is it mere temporal succession? Is it an inherent conceptual connection between two things?  Is it objective?  Is it subjective?  Are causes simultaneous with effects?  Is it a one to one correspondence?  etc. etc. etc.

Quote:
I don't know, seems to me a lot of scientists address the nature of causality very regularly.

No, they do not.  They observe physical events happening in succession, and they create a model based on their observations.  They do not care about the aforementioned issues; they are merely concerned with whether the physical behavior is repeatable and what we may infer about the universe based on that repeated physical behavior.  At the philosophical level of explanation, scientists have not addressed anything--and they shouldn't because admittedly they are not philosophers.  

If you disagree with me, then please produce some sort of peer reviewed publication regarding scientific observations of causation.  Is there a law of causation in science telling us precisely what causation is, whether causes are greater than effects, and so forth?  If so, who put it together?  As far as I can tell, scientists *presuppose* causality in order to go forward with their methodology--in other words, they leave the philosophizing to the philosophers.

Quote:
Why do we have earthquakes, why do train lights look red as the train passes us, why do we have volcanoes, why do we have fossils of creatures that no one has ever seen alive, why is the climate changing, why, why, why, why.......

This has nothing to do with metaphysical issues regarding causes and effects.

Quote:
Qualitative perfections of individual causes and effects?  They are what they are - no perfection required or needed.

That does not even make sense--I take it you did not understand what I meant.  

 

 

Why would anyone even care about those questions 'metaphysically', how are they even relevant to our experience?  It seems like asking a non-sense question to me...what's the point?  What problem exists that these questions need to find an answer for?  What powers of explanation do non-materialistic answers provide?

 

Thank you, that was the point I was trying to make but it went over Mr. M's head, obviously.

 

-- I feel so much better since I stopped trying to believe.

"We are entitled to our own opinions. We're not entitled to our own facts"- Al Franken

"If death isn't sweet oblivion, I will be severely disappointed" - Ruth M.